Which of the following is an equation that involves a variable under a radical sign?

Quadratic Equation
Radical Eqaution
Exponential Equation
Linear Equation

Answers

Answer 1
Radical equation i believe would be the answer.

Related Questions

what came in the mail today

Answers

Answer:

me

Step-by-step explanation:

A bunch of bills for spectrum tv

Lex be a random variable that represents the pH of arterial plasma (le, acidity of the blood). For healthy adults, the mean of the distribution is - 7.4.1 A new drug for arthritis has
been developed. However, it is thought that this drug may change blood pH. A random sample of 31 patients with arthritis took the drug for 3 months. Blood tests showed that x = 8.6 with
sample standard deviations - 2.7. Use a 5% level of significance to test the claim that the drug has changed (either way) the mean pH level of the blood.
USE SALT
(a) what is the level of significance?

what is the value of the sample test sisse? (Round your answer to three decimal places)

Answers

Answer:

2.304

Step-by-step explanation:

plz help me find x!!!!!!!!!!!!!!!!

Answers

Answer:

A: 45

B: 110

C: 25

Step-by-step explanation:

all angles of a triangle always add together to make 180

x + 25 + x + 65 = 180

combine like terms

2x + 90 = 180

subtract 90 from both sides of the equation

2x = 180 - 90

divide both sides of the equation by 2

x = 90/2

x = 45

A group of students formed a circle during a game. The circumference of the circle was about 43.96 feet, and the diameter of the circle was 14 feet.



Which expression best represents the value of
π
?

A.
43.96
7

B.
43.96
14

C.
7
43.96

D.
14
43.96

Answers

Answer:

pi = pi

Step-by-step explanation:

Circumference = 2 * pi * r

Equation you need:

43.96 = 2 * pi * 7

43.96 = 2 * 7 * pi

43.96 / 14 = pi

pi = 3.14

can someone please help me

Answers

64 because 4 turns into -8 -8 turns into 16 and 16 turns into -32 so it must be 64

64 is the right answer

Which of the following is a radical equation?
a. x StartRoot 3 EndRoot = 13
b. x + StartRoot 3 EndRoot = 13
c. StartRoot x EndRoot + 3 = 13
d. x + 3 = StartRoot 13 EndRoot

Answers

The starting radical equation of the problem is d x+3=StarRoot 13 EndRoot

Answer:

the answer is D

Step-by-step explanation:

Keith 1,342 cupcake rounds to 1,300 and joe 1,347 cupcake rounds to 1,350 who round to the nearest hundred?

Answers

Answer:

Keith

Step-by-step explanation:

When you are rounding to the nearest hundred it would NOT include a tens or ones value. So in this case 1,300 would be correct.

Hope that helps and have a great day!

7th grade math help me pleasee

Answers

Answer:

a, c, b

Step-by-step explanation:

ITS CORRECT TRUST MEEE

-21x^2+35x=0
Solve for x

Answers

Answer:

Step-by-step explanation:

exact form: x=63/5

help me please
*I select an answer, but I don't know if it's right or wrong*​

Answers

Answer:

yes your answer it right.... brainliest plsss

first pirson to type 1 i will see if i can give brainlist but idk how bot i will try

Answers

Answer:

1

Step-by-step explanation:

you asked to type 1

help me solve please.​

Answers

X=1/2 because if you set the equations equal to each other you would get that x-value


The fraction 24/28 In simplest form is____.
12/14
48/56
6/9
6/7

Answers

Answer:

6/7

Step-by-step explanation:

Answer:

6/7

Step-by-step explanation:

divide it by 4

Calculating Rate of change

Answers

Answer/Step-by-step explanation:

We are given the following coordinates of two points on the line of the graph shown in the question as: A(2, 1) and B(4, 2)

Vertical change from point A = [tex] y_2 - y_1 = 2 - 1 = 1 [/tex]

Horizontal change from point A = [tex] x_2 - x_1 = 4 - 2 = 2 [/tex]

Rate of change = [tex] \frac{y_2 - y_1}{x_2 - x_1} = \frac{1}{2} = 0.5 [/tex]

Consider 8 = − 2/3x. 1 Which is the BEST first step to take when solving the given equation?


A) Multiply each side by 3/2.

B) Multiply each side by −3/2.

C) Multiply each side by −2/3.

D) Add 2/3x to each side.


Solve the equation.

A) x = 10

B) x = 6

C) x = −10

D) x = −12

Answers

Solve for x by simplifying both sides of the equation, then isolating the variable. Answer x = −12

Help me pleaseeeeee!!

Answers

The answer : 4.159%

How : 12890000 divided by 309.9 million gives you - 4.159%

what is 1555 times 67

Answers

Answer:

104185

Step-by-step explanation:

Answer:

104,185

Step-by-step explanation:

HELP! A line goes through the points (8,10) and .

What is the slope of the line? Show your work

Write the equation of the line in point-slope form. Show your work

Write the equation of the line in slope-intercept form. Show your work.


I will give y'all 50 points please answer and show your work! Please keep in mind I am not afraid to report someone.

Answers

Answer:

slope is [tex]\frac{3}{5}[/tex] ; slope-intercept form is y = [tex]\frac{3}{5}[/tex] x + [tex]\frac{13}{20}[/tex] ; point-slope form is 4 - 10 = [tex]\frac{3}{5}[/tex] (-2 - 8)

Step-by-step explanation:

first, we find the slope by using our points (8,10) and (-2,4)

to do so, you use the expression [tex]\frac{y1 - y2}{x1 - x2\\}[/tex] where (8,10) is point 1 and (-2,4) is point 2

this results in [tex]\frac{10-4}{8--2}[/tex]

10-4 is 6 and 8--2 is 10

which results in the slope of [tex]\frac{3}{5}[/tex]

now that we have the slope, we can solve for the y-intercept using the slope-intercept formula [y = mx +b]

we choose a point [im using (8,10)] and but the values in for x and y in the equation, which gives us 10 = 3/5 (8) + b , and solve for b

to solve for b, we start by dividing both sides by 8, resulting in 5/4 = 3/5 +b

then, subtract 3/5 from 5/4, resulting in 13/20 = b

To check the work, we use slope-intercept formula again [yeyyy]

y = 3/5x + 13/20 ; 4 = 3/5 (-2) + 13/20

i used my calculator to check, it's correct

SO: the slope-intercept form of the equation is y = [tex]\frac{3}{5}[/tex] x + [tex]\frac{13}{20}[/tex]

now we get to do the point-slope formula [yeyyyyyyyyyyy!!!]

point-slope formula is y - y1 = m (x - x1)

the slope [m] is [tex]\frac{3}{5}[/tex]

and we use any given point [im using (8,10) again, but any point should technically be correct] for y1 and x1

inputting our values, we get y - 10 = 3/5 (x - 8)

now, you use any other point for x and y to check the work

4 - 10 = 3/5 (-2 - 8)

-2 - 8 is -10, and 3/5 times -10 is -6, which is equal to 4 - 10

Answer:

3/5

Step-by-step explanation:

did the test


Please help,
Determine which is the graph of the given function. f(x) 2x if < - 2 2x - 2 if - 2 < x < 2 -2 if x > 2​

Answers

Answer:

C

Step-by-step explanation:

Answer:

C

Step-by-step explanation:

Angle b= 61, angle c=57, and =2x what does x =

Answers

Answer:

can you provide more information

Step-by-step explanation:

A line contains points M(1, 3) and N(5, 0). What is the slope of MN?

Answers

Answer:

-3/4

Step-by-step explanation:

y2-y1/x2-x1

0-3/5-1

A line contains points M(1, 3) and N(5, 0). The slope of MN is -3/4.

What is the slope of the line?

A line's slope provides information on the steepness and direction of the line. By calculating the difference between the coordinates of the two locations, (x1,y1) and (x2,y2), it is simple to calculate the slope of a straight line between them.

The slope, m, of the line, is calculated through the equation,

The formula used to find the slope is,

m = (y2 - y1)/ (x2 - x1)

Where (x1, y1) and (x2, y2) are the points

The given points on the line are:

M(1, 3) and N(5, 0)

Substituting it in the formula,

m = (0 - 3)/ (5 - 1)

m = -3/4

Therefore, the slope of MN is -3/4.

Learn more about the slope of the line here:

https://brainly.com/question/14511992

#SPJ7

It cost Emma $5.34 to send 178 text messages. How much does each text cost to send?

Answers

Answer:

approximately $0.33

Step-by-step explanation:

you just do 178/5.34

Skyler solved an equation incorrectly, as shown below:

Step 1: 12 + x = 36
Step 2: x = 36 + 12
Step 3: x = 24

Which statement best explains why Step 2 is incorrect in Skyler's solution?

Answers

Answer:

Step 2 is incorrect.

Step-by-step explanation:

It's incorrect because I'm pretty you need to do inverse operation. To get 24 you must subtract, not add.

10b - 2 = 7b - 7.4 NEED ASAP!!!!!!!

Answers

Answer:

b=−1.8

Step-by-step explanation:

Graph the following figure in the coordin
X(0,1), Y(4, - 2), Z(-5,-2)
The perimeter is

Answers

Answer: I am in highschool and im doing this stuff. If you find out the answer let me know please.

A student uses the following steps to prove the sine sum identity. The proof is incorrect. Which steps is incorrect

Answers

Answer:

4

Step-by-step explanation:

The incorrect step is Step 4, cos (π/2 -x) cos(-y) + sin (π/2 -x)sin(-y), the correct option is C.

What is Trigonometric Identity?

Trigonometric Identities are based on the trigonometric ratios used to solve Trigonometric equations.

The steps used to solve the trigonometric identity are :

Step 1 :  sin(x+y)

Step 2: cos ((π/2) - (x+y))

Step 3 : cos ( (π/2 -x) -y)

Step 4 : cos (π/2 -x) cosy + sin (π/2 -x)siny

Step 5: sinxcosy+cosxsiny

Therefore,the correct step is cos (π/2 -x) cosy + sin (π/2 -x)siny  and not cos (π/2 -x) cos(-y) + sin (π/2 -x)sin(-y) , the incorrect step is Step 4.

The complete question is attached with the answer.

To know more about Trigonometric Identity

https://brainly.com/question/12537661

#SPJ2

help!!!

3 Peter measured the width of a room as 28 feet
and the length as 17 feet. About how many
square feet does the room cover?
Area = length X width

Answers

answer: 476
Area = length X width 28x17= 476

The midpoint of a line segment is (−3,2). One endpoint of the line segment is (4,−7).

What are the coordinates of the other endpoint of the line segment?

Answers

Answer:

The other endpoint is  (9, −6)

Step-by-step explanation:

(x1,y1)=(3,2) and M=(6,−2)

x1+x22=x     and     y1+y22=y

3+x22=6     and     2+y22=−2     ←×2

3+x2=12     and     2+y2=−4

x2=9              and     y2=−6

The coordinates of the other endpoint of the line segment are [tex]Y(x,y) = (-10, 11)[/tex].

Vectorially speaking, the midpoint of a line segment is determined by the following expression:

[tex]M(x,y) = \frac{1}{2}\cdot X(x,y) + \frac{1}{2}\cdot Y(x,y)[/tex] (1)

Where:

[tex]M(x,y)[/tex] - Midpoint.[tex]X(x,y)[/tex], [tex]Y(x,y)[/tex] - Endpoints of the line segment.

If we know that [tex]M(x,y) = (-3, 2)[/tex] and [tex]X(x,y) = (4, -7)[/tex], then the coordinates of the other endpoint is:

[tex]\frac{1}{2}\cdot Y(x,y) = M(x,y) -\frac{1}{2}\cdot X(x,y)[/tex]

[tex]Y(x,y) = 2\cdot M(x,y) -X(x,y)[/tex]

[tex]Y(x,y) = 2\cdot (-3,2)-(4,-7)[/tex]

[tex]Y(x,y) = (-6, 4) - (4, -7)[/tex]

[tex]Y(x,y) = (-10, 11)[/tex]

The coordinates of the other endpoint of the line segment are [tex]Y(x,y) = (-10, 11)[/tex].

We kindly invite to check this question on midpoints: https://brainly.com/question/17506315

PLZ HELP THIS IS DUE SOON!!!!!!! Divide. Round to the nearest hundredth if necessary.
To the nearest hundredth, 80.88÷8.425 is_______

Answers

Answer:

10

Step-by-step explanation:

when you divide 80.88 by 8.425 you get 9.6

and since youre rounding, you can round up 6 because its higher than 5

and your final answer should be 10.

hoped this helped! :)

Which set of ordered pairs does not represent a function?

Group of answer choices

{(6, 0), (-9, 5), (1, -5), (0, 0)}

{(4, 6), (6, 8), (3, 2), (-6, 8)}

{(-9, -7), (-8, -7), (8, 2), (9, -5)}

{(-8, 6), (-8, 5), (-9, 4), (-5, 3)}

Answers

Answer:

the last one

Step-by-step explanation:

No points on a function can have the same x value, can you give me brainliest

Other Questions
What is the volume of the shape below A student opens a recently purchased software package and begins reading the enclosed materials. What information might be included in the software license? Check all that apply. What are some of the similarities between the American and French Revolutions? HELPPS PLEASE FOR BRIANILIST Pick your best idea and make a blueprint, detailed drawing, or you may describe it in a paragraph. (Sketch your blue prints here.) what does it mean by that please help me What does the Declaration of Independence document state the people have the right to dowhen they have a bad government? Summarize the battle of pea ridge and its outcome? How does Grant Wood share stories in his artworks? How many significant figures are in the number: 800 can i have 300mn + 500mn =_________________________________________________________________________________________________i need help 1. Which of the following weatherconditions support the development ofa hurricane? Change the sentence from affirmative to negative :-n1. You are richer than me. 2. She tried all plans.3. A few farmers saved their crop.eg:-You are richer than I. I am not richer than you. Its meaning is 'you' is richer than 'I'. The sentence was changed into negative sentence:I am not richer than you. The meaning of the sentence is the same; 'You' is richer than 'I'. i bring tea in active voice Helppppp! I dot know the answer Coupon collector is a classic statistic problem with many practical applications. The problem is to pick objects from a set of objects repeatedly and determine how many picks are needed for all the objects to be picked at least once. A variation of the problem is to pick cards from a shuffled deck of 52 cards repeatedly and find out how many picks are needed before you see one of each suit. Assume a picked card is placed back in the deck before picking another. Write a program to simulate the number of picks needed to get total of four cards from each different suit and display the four cards picked (it is possible that a card may be picked twice). Here is a sample run of the program:4 of Diamonds8 of SpadesQueen of Clubs8 of HeartsNumber of picks: 9 Our solar system is in the _______ arm of the Milky Way galaxy, 28,000 light years from its center. Which statement best supports the fact that the United States is becoming more diverse?Quotas restricting immigration from certain parts of the world have been removed, but there are still limits onimmigrationO By 2050, more than half the population of the United States will belong to an ethnic minority group.O Between 2007 and 2009, the number of illegal immigrants in the United States dropped from nearly 12 millionless than 11 million.O More people in the United States are of Latin American ancestry than of Asian ancestry. Terrance lives in an area that is expensive, but he does not want to move. When looking at a career, what should Terrance consider? He should choose a career cluster that will allow him to support himself. He should pick a career that speaks to his heart and worry about the finances later. He should ask his family to help support him. He should not worry because he can dictate how much money he will earn. how are population sizes kept? write an expression for the area of the following quadrilateral with the fewest terms. all sides are congruent. ALSO PLEASE SHOW ME HOW YOU GOT IT